Difference between revisions of "2021 JMPSC Invitationals Problems/Problem 11"
(→Solution) |
(→Solution) |
||
Line 4: | Line 4: | ||
==Solution== | ==Solution== | ||
First note that the integers in the given arithmetic progression are precisely the integers which leave a remainder of <math>4</math> when divided by <math>5</math>. | First note that the integers in the given arithmetic progression are precisely the integers which leave a remainder of <math>4</math> when divided by <math>5</math>. | ||
+ | |||
Suppose a perfect square <math>m^2</math> is in this arithmetic progression. Observe that the remainders when <math>0^2</math>, <math>1^2</math>, <math>2^2</math>, <math>3^2</math>, and <math>4^2</math> are divided by <math>5</math> are <math>0</math>, <math>1</math>, <math>4</math>, <math>4</math>, and <math>1</math>, respectively. Furthermore, for any integer <math>m</math>, <cmath>(m+5)^2 = m^2 + 10m + 25 = m^2 + 5(2m + 5),</cmath> and so <math>(m+5)^2</math> and <math>m^2</math> leave the same remainder when divided by <math>5</math>. It follows that the perfect squares in this arithmetic progression are exactly the numbers of the form <math>(5k+2)^2</math> and <math>(5k+3)^2</math>, respectively. | Suppose a perfect square <math>m^2</math> is in this arithmetic progression. Observe that the remainders when <math>0^2</math>, <math>1^2</math>, <math>2^2</math>, <math>3^2</math>, and <math>4^2</math> are divided by <math>5</math> are <math>0</math>, <math>1</math>, <math>4</math>, <math>4</math>, and <math>1</math>, respectively. Furthermore, for any integer <math>m</math>, <cmath>(m+5)^2 = m^2 + 10m + 25 = m^2 + 5(2m + 5),</cmath> and so <math>(m+5)^2</math> and <math>m^2</math> leave the same remainder when divided by <math>5</math>. It follows that the perfect squares in this arithmetic progression are exactly the numbers of the form <math>(5k+2)^2</math> and <math>(5k+3)^2</math>, respectively. | ||
− | Finally, the sequence of such squares is <cmath>(5\cdot 0 + 2)^2 | + | |
+ | Finally, the sequence of such squares is <cmath>(5\cdot 0 + 2)^2, (5\cdot 0 + 3)^2, (5\cdot 1 + 2)^2, (5\cdot 1 + 3)^2,\cdots.</cmath> | ||
+ | |||
+ | In particular, the first and second such squares are associated with <math>k=1</math>, the third and fourth are associated with <math>k=2</math>, and so on. It follows that the <math>37^{\text{th}}</math> such number, which is associated with <math>k=18</math>, is <cmath>(5\cdot 18 + 2)^2 = 92^2 = 9409.</cmath> | ||
+ | |||
+ | Therefore the arithmetic progression must not reach <math>8464</math>. This means the desired answer is <math>\boxed{8459}.</math> ~djmathman |
Revision as of 14:49, 11 July 2021
Problem
For some , the arithmetic progression has exactly perfect squares. Find the maximum possible value of
Solution
First note that the integers in the given arithmetic progression are precisely the integers which leave a remainder of when divided by .
Suppose a perfect square is in this arithmetic progression. Observe that the remainders when , , , , and are divided by are , , , , and , respectively. Furthermore, for any integer , and so and leave the same remainder when divided by . It follows that the perfect squares in this arithmetic progression are exactly the numbers of the form and , respectively.
Finally, the sequence of such squares is
In particular, the first and second such squares are associated with , the third and fourth are associated with , and so on. It follows that the such number, which is associated with , is
Therefore the arithmetic progression must not reach . This means the desired answer is ~djmathman